Another mod related question

This topic has expert replies
User avatar
Senior | Next Rank: 100 Posts
Posts: 53
Joined: Thu Dec 25, 2014 2:23 pm
Thanked: 1 times

Another mod related question

by utkalnayak » Tue Jan 27, 2015 5:37 pm
Is x > 1?
1. (x+1)(|x|-1) > 0
2. |x| < 5

OA: A

OK, understand that 2 is definitely helpful, could you please breakdown option 1 please?
Thanks,
Utkal

User avatar
GMAT Instructor
Posts: 15539
Joined: Tue May 25, 2010 12:04 pm
Location: New York, NY
Thanked: 13060 times
Followed by:1906 members
GMAT Score:790

by GMATGuruNY » Tue Jan 27, 2015 8:05 pm
utkalnayak wrote:Is x > 1?
1. (x+1)(|x|-1) > 0
2. |x| < 5
Statement 1: (x+1)(|x|-1) > 0
Determine the CRITICAL POINTS: the values where (x+1)(|x|-1) = 0.
(x+1)(|x|-1) = 0 when x=-1 or x=1.
To determine the ranges where (x+1)(|x|-1) > 0, test one value to the left and right of each critical point.

x<-1:
If we plug x=-2 into (x+1)(|x|-1) > 0, we get:
(-2+1)(|-2|-1) > 0
(-1)(1) > 0
-1 > 0.
Doesn't work.
Thus, x<-1 is not a valid range.

-1<x<1:
If we plug x=0 into (x+1)(|x|-1) > 0, we get:
(0+1)(|0|-1) > 0
(1)(-1) > 0
-1 > 0.
Doesn't work.
Thus, -1<x<1 is not a valid range.

x>1:
If we plug x=2 into (x+1)(|x|-1) > 0, we get:
(2+1)(|2|-1) > 0
(3)(1) > 0
3 > 0.
This works.
Thus, x>1 is a valid range.

Since x>1 is the only valid range, (x+1)(|x|-1) > 0 implies that x>1.
SUFFICIENT.

Statement 2: |x| < 5
It's possible that x=2, in which case x>1.
It's possible that x=0, in which case x<1.
INSUFFICIENT.

The correct answer is A.
Private tutor exclusively for the GMAT and GRE, with over 20 years of experience.
Followed here and elsewhere by over 1900 test-takers.
I have worked with students based in the US, Australia, Taiwan, China, Tajikistan, Kuwait, Saudi Arabia -- a long list of countries.
My students have been admitted to HBS, CBS, Tuck, Yale, Stern, Fuqua -- a long list of top programs.

As a tutor, I don't simply teach you how I would approach problems.
I unlock the best way for YOU to solve problems.

For more information, please email me (Mitch Hunt) at [email protected].
Student Review #1
Student Review #2
Student Review #3

User avatar
Legendary Member
Posts: 2663
Joined: Wed Jan 14, 2015 8:25 am
Location: Boston, MA
Thanked: 1153 times
Followed by:128 members
GMAT Score:770

by DavidG@VeritasPrep » Tue Jan 27, 2015 8:13 pm
As a general rule, when I see a setup where S1 is very complicated, and S2 is so obviously not sufficient that I can evaluate it in my head in three seconds, I become very very suspicious that the more complicated statement will be sufficient on its own.

Is x > 1?
1. (x+1)(|x|-1) > 0

The algebra looks hairy here, so I'd rather pick numbers. Say x = 2. That satisfies the statement, so clearly, I can get a YES to the question. Now the question is can I get a NO? Well, if x = 1/2, the product will be negative, so that won't work. And If x = -2, the product is still negative. Test away, and you'll see that only numbers bigger than one will work here, so S1 is sufficient. (Essentially, if -1 < x < 1 the first term will be positive and the second term will be negative, yielding a negative product. And if x < -1, the first term will be negative and the second term will be positive, again yielding a negative product. So then x is going to have to be bigger than 1.)


2. |x| < 5
x = 3 or x = 0; Not sufficient; (and this statement insulted our collective intelligence.)

User avatar
Legendary Member
Posts: 2131
Joined: Mon Feb 03, 2014 9:26 am
Location: https://martymurraycoaching.com/
Thanked: 955 times
Followed by:140 members
GMAT Score:800

by MartyMurray » Wed Jan 28, 2015 4:12 am
The one thing I would make clear is that for Statement 1 to work, both left side terms have to be either negative or positive, and neither can be 0.

So when you are plugging in, you don't need to actually solve the whole thing. You just need to see if you are creating either two positive terms or two negative terms. If you are not, then the number you are using doesn't work. If you are, then it does.
Marty Murray
Perfect Scoring Tutor With Over a Decade of Experience
MartyMurrayCoaching.com
Contact me at [email protected] for a free consultation.

GMAT Instructor
Posts: 2630
Joined: Wed Sep 12, 2012 3:32 pm
Location: East Bay all the way
Thanked: 625 times
Followed by:119 members
GMAT Score:780

by Matt@VeritasPrep » Mon Feb 09, 2015 2:26 am
Here's another algebraic approach.

Since |x| = √x², S1 really gives us

(x + 1)(√x² - 1) > 0

√x² * (x + 1) - 1 * (x + 1) > 0

√x² * (x + 1) > (x + 1)

At this point, we know that √x² is not 0 or 1. (If it were, both sides would be equal, but we're told that the left hand side is greater.) We thus have two cases to consider.

Case 1:: (x + 1) > 0

In this case, we divide both sides of the equation to get √x² > 1. This reduces to x > 1 or -1 > x, but the second case is impossible. (It would contradict (x + 1) > 0, i.e. x > -1, which we assumed to get here in the first place.)

Case 2:: 0 > (x + 1)

In this case, we divide both sides of the equation to get √x² < 1. This reduces to -1 < x < 1, but this is also impossible, since it contradicts 0 > (x + 1), or - 1 > x. Hence Case 2 is entirely impossible.

So the ONLY possibility is x > 1, and S1 is sufficient.